A(3, 7), B(5, 7), C(3-7), D(5, -7)
what is the area ?

Answers

Answer 1

Answer:

28 square units.

Step-by-step explanation:

This is a rectangle with sides (7 - (-7) and (5 - 3)

= 14 by 2

= 28 unit^2.


Related Questions

Robert is putting new roofing shingles on his house. Each shingle is 1 2/3 feet long. The north part of the house has a roof line that is 60 feet across. How many shingles can be placed (side by side) on the north part of the house?

Answers

Answer: 36 shingles can be placed on the north part of the house.

Step-by-step explanation:

Given: Length of each shingle = [tex]1\dfrac23[/tex] feet = [tex]\dfrac53[/tex] feet.

The north part of the house has a roof line that is 60 feet across.

Then, the number of  shingles can be placed  on the north part of the house = (Length of roof line in north part) ÷ (Length of each shingle)

[tex]=60\div \dfrac{5}{3}\\\\=60\times\dfrac{3}{5}\\\\=12\times3=36[/tex]

Hence, 36 shingles can be placed on the north part of the house.

The difference of two trinomials is x2 − 10x + 2. If one of the trinomials is 3x2 − 11x − 4, then which expression could be the other trinomial? 2x2 − x − 2 2x2 + x + 6 4x2 + 21x + 6 4x2 − 21x – 2

Answers

Answer: [tex]4x^2-21x-2[/tex] .

Step-by-step explanation:

Given: The difference of two trinomials is [tex]x^2 -10x + 2[/tex]. If one of the trinomials is [tex]3x^2 - 11x - 4[/tex].

Then, the other trinomial will be ([tex]3x^2 - 11x - 4[/tex] ) + ([tex]x^2 -10x + 2[/tex])

[tex]=3x^2-11x-4+x^2-10x+2\\\\=3x^2+x^2-11x-10x-4+2\\\\=4x^2-21x-2[/tex]

Hence, the other trinomial could be : [tex]4x^2-21x-2[/tex] .

Please help meeee
Find the value of d.
A. 47
B. 56
C. 75
D. 30

Answers

Answer:

56

Step-by-step explanation:

Angle Formed by Two Chords= 1/2(sum of Intercepted Arcs)

75 = 1/2 (94+d)

Multiply by 2

75*2 = 94+d

150 = 94+d

Subtract 94 from each side

150-94 = d

56=d

Examine the two triangles. Are the triangles congruent? Justify your conclusions. If they are congruent, complete the following statement: "Yes, triangle __ congruent to triangle __ giving a detailed explanation of your reasoning. If they are not congruent, explain why you think so. Be specific in your answer and make sure to show your work.

Answers

Answer: The triangles are not congruent

==========================================

Explanation:

For triangle DEF, the missing angle D is

D+E+F = 180

D+80+60 = 180

D+140 = 180

D = 180-140

D = 40

While the missing angle K in triangle JKL is

J+K+L = 180

80+K+50 = 180

K+130 = 180

K = 180-130

K = 50

---------------------

The three angles for triangle DEF are

D = 40E = 80F = 60

The three angles for triangle JKL are

J = 80K = 50L = 50

We don't have all the angles matching up. We need to have the same three numbers (the order doesn't matter) show up for both triangles in order for the triangles to be congruent. This is because congruent triangles have congruent corresponding angles.

The only pair that matches is E = 80 and J = 80, but everything else is different. So there is no way the triangles are congruent.

Notice how triangle JKL has two congruent base angles (K = 50 and L = 50), so this triangle is isosceles. Triangle DEF is not isosceles as we have three different angles, so this triangle is scalene.

Find the surface area of the triangular prism.

Answers

Answer:

169 [tex]cm^{2}[/tex]

Step-by-step explanation:

Surface area (SA) = 2B + PH

                       SA = 2 ([tex]\frac{1}{2}[/tex] x 9 x 6) + (7+7+9) 5

                             = 2 (27) + (23) 5

                             = 54 + 115

                        SA = 169 [tex]cm^{2}[/tex]

Musah stands at the center of a rectangular field. He first takes 50 steps north, then 25 steps west and finally 50 steps on a bearing of 315°. Sketch musah's movement. How far west is musah's final point from the center?

Answers

Answer:  4.17 steps

Step-by-step explanation:

Draw a point to use as the center and then sketch 50 units north (up) and 25 units west (left) and 315° which creates a right triangle that has an angle of 360° - 315° = 45°

Use Pythagorean Theorem to find the length of the hypotenuse.

50² + 25² = hypotenuse²   --> hypotenuse = 55.9 units

Since Musah only walked 50 units along the hypotenuse, he is 5.9 units from the center.

Create another right triangle using the remaining 5.9 units as the hypotenuse.  You can use 45°-45°-90° rules OR sin 45° to find the horizontal distance from the center to be 4.17.

see attachment for sketch

1. (a) Find the probability that a 90% free-throw shooter makes 10 consecutive free-throws, assuming that individual shots are independent.

Answers

Answer:

[tex]Probability = 0.35[/tex]

Step-by-step explanation:

Given

Probability of success free throw = 90%

Number of throw = 10

Required

Determine the probability of 10 consecutive free throws

Let p represents the given probability

[tex]p = 90\%[/tex]

Convert to decimal

[tex]p = 0.9[/tex]

Let n represents the number of throw

[tex]n = 10[/tex]

Provided that each throw is independent;

The probability of n consecutive free throw is

[tex]p^n[/tex]

Substitute 0.9 for p and 10 for n

[tex]Probability = 0.9^{10}[/tex]

[tex]Probability = 0.3486784401[/tex]

[tex]Probability = 0.35[/tex] (Approximated)

Find the cosine of the angle between the planes x+y+z=0 and 4x+3y+z=1.

Answer: cosθ=

Answers

The angle between the planes is the same as the angle between their normal vectors, which are

n = ⟨1, 1, 1⟩

n = ⟨4, 3, 1⟩

The angle θ between the vectors is such that

⟨1, 1, 1⟩ • ⟨4, 3, 1⟩ = ||⟨1, 1, 1⟩|| ||⟨4, 3, 1⟩|| cos(θ)

Solve for cos(θ) :

4 + 3 + 1 = √(1² + 1² + 1²) √(4² + 3² + 1²) cos(θ)

8 = √3 √26 cos(θ)

cos(θ) = 8/√78

The area of the circle x² + y2 - 6x-4y +9 = 0 is​

Answers

Answer:

Your answer is here.Enjoy dude

Answer:

12.56 unit²

Step-by-step explanation:

Given:x² + y² - 6x - 4y + 9 = 0To find:The area of circleSolution:

The form of the circle is:

(x- h)² + (y-k)² = r²

Let's bring the given to the form of a circle as above:

x² + y² - 6x - 4y + 9 = 0x² - 6x  + y²-  4y + 9 = 0        ⇒ combining like terms and completing squarex² - 6x + 9 + y²- 4y + 4 = 4    ⇒ adding 4 to both sides(x-3)² + (y - 2)² = 2²                ⇒ got the form of this circle

As per the form, we got r² = 2², so the radius of circle is 2 units.

The area of circle:

A= πr² = 3.14×2² = 12.56 unit²

A museum curator is hanging 7 paintings in a row for an exhibit. There are 4 Renaissance paintings and 3 Baroque paintings. From left to right, all of the Renaissance paintings will be hung first, followed by all of the Baroque paintings. How many ways are there to hang the paintings

Answers

Answer:

144 ways

Step-by-step explanation:

Number of paintings = 7

Renaissance = 4

Baroque = 3

We are hanging from left to right and we will first hang Renaissance painting before baroque painting.

For Renaissance we have 4! Ways of doing so. 4 x3x2x1 = 24

For baroque we have 3! Ways of doing so. 3x2x1 = 6

We have 4!ways x 3!ways

= (4x3x2x1) * (3x2x1) ways

= 144 ways

Therefore we have 144 ways to hang the painting.

what is the prime factorization of 55^5 x 65 x 9^15 and why? A. 3^15 * 5^6 *11^5*13 B. 3^30 *5^6 *11^5 *13 C.3^30 * 5^6 *11 * 13 D. 3^30 *5^5*11^5*13

Answers

Answer:

B

Step-by-step explanation:

1. Lets focus on 55^5

55^5 = 11^5 * 5^5

2. now on 65

65 = 5 * 13

3. now on 9^15

9^15=(3^2)^15 = 3^30

4. combine all three parts

11^5 * 5^5 * 5 * 13 * 3^30 = 11^5*5^6*13*3^30

so our answer is B

(x−1)(x−7)=0 PLEASE HELP

Answers

Answer:

1, 7

Step-by-step explanation:

Because the product is 0, either (x-1) or (x-7) is equal to 0. That means that x = 1, or 7

What Number is equivalent to 4^3
A. 7
B. 12
O C. 64
D. 81

Answers

Answer:

C

Step-by-step explanation:

4³ means 4 multiplied by itself 3 times, that is

4 × 4 × 4

= 16 × 4

= 64 → C


The length of a rectangle is 4 units more than its width. The area of the rectangle is 25 more than 4 times the
width. What is the width of the rectangle?
A А 9
B -5
С. 3
D 5
Please select the best answer from the choices provided

Answers

Answer:

D

Step-by-step explanation:

let the width = w

w = w

L = 4 + w

Area = 4*w + 25 = L * w

4w + 25 = L * w                  Substitute for the length

4w + 25 = (w*(w + 4))

4w + 25 = w^2 + 4w          Subtract 4w from each side

w^2 = 25                            Take the square root.

w = +/- 5

- 5 has no meaning.

w = 5

IQ tests are scaled so that the mean score in a largepopulation should be μ =100. We suspect that the very-low-birth-weight population has mean score less than100. Infants weiging less than 1500 grams at birth are classed as "very low birth weight". Low birth weight carriesmany risks. One study followed 113 male infants with very low birth weight to adulthood. At age 20, the mean IQ score for these men was (x bar=87.6.) Iq scores vary Normally with standard deviation σ=15. Give a 95% confidence interval for the mean IQ score at age 20 for allvery-low-birth-weight males. Use the four-step process for confidence interval.

Answers

Answer:

The 95% confidence interval is  [tex]84.83< \mu < 90.37[/tex]

Step-by-step explanation:

From the question we are told that

    The  sample size is  [tex]n = 113[/tex]

     The sample mean is  [tex]\= x = 87.6[/tex]  

      The standard deviation is  [tex]\sigma = 15[/tex]

     

Given that the confidence level is  95% then the level of significance is mathematically represented as

            [tex]\alpha = 100 - 95[/tex]

             [tex]\alpha = 5\%[/tex]

             [tex]\alpha = 0.05[/tex]

Next we obtain the critical value of  [tex]\frac{\alpha }{2}[/tex] from the normal distribution table, the value  is  

              [tex]Z_{\frac{ \alpha }{2} } = 1.96[/tex]

Generally the margin of error is mathematically evaluated as

              [tex]E = Z_{\frac{\alpha }{2} } * \frac{ \sigma}{ \sqrt{n} }[/tex]

=>          [tex]E = 1.96 * \frac{ 15}{ \sqrt{113} }[/tex]

=>          [tex]E = 2.77[/tex]

The  95% confidence interval is mathematically represented as

          [tex]\= x - E < \mu < \= x + E[/tex]

substituting values

        [tex]87.6 - 2.77< \mu < 87.6 + 2.77[/tex]

        [tex]84.83< \mu < 90.37[/tex]

NO LINKS OR ANSWERING QUESTIONS YOU DON'T KNOW!! Chapt 15 part 2a

Explain the process of matrix multiplication. What are the dimensions 9f of the resulting matrix? Use the following matrices for your explanation of the process. AB. Show your work. ​

Answers

We are given the matrices A and B

[tex]A = \left[\begin{array}{ccc}2&3&-1\\0&2&5\\2&4&0\end{array}\right][/tex]

[tex]B = \left[\begin{array}{ccc}2\\1\\2\end{array}\right][/tex]

Multiplying these matrices:

We multiply matrices by taking the first column of the first matrix and the first row of the second matrix

we will multiply all the terms of the first column of the first matrix and multiply them by the terms of the first row of the second matrix, one by one

[tex]AB = \left[\begin{array}{ccc}2(2) + 3(1) + -1(2)\\0(2) + 2(1) + 5(2)\\2(2) + 4(1) + 0(2)\end{array}\right][/tex]

[tex]AB = \left[\begin{array}{ccc}5\\12\\8\end{array}\right][/tex]

9514 1404 393

Explanation:

Two matrices with dimensions (numbers of (rows, columns)) of (a, b) and (c, d) can only be multiplied if the number of columns in the left matrix is equal to the number of rows in the right matrix. That is, b=c. The dimensions of the product matrix will be (a, d).

For row i of the left matrix and column j of the right matrix, element a(i,j) of the product matrix is the dot-product of row i with column j. (The dot-product of two vectors is the sum of the products of corresponding elements.)

__

The example matrices have (row, column) dimensions (3, 3) and (3, 1), so can be multiplied with a result having dimensions (3, 1).

It is useful to refer to an element of a matrix by specifying the row and column in which it resides. An element of matrix 'A' in row 2 and column 3 can be referred to as A(2,3). Often, subscripts are used, as in ...

  [tex]A_{i,j}[/tex]

For matrix C = A·B, the element C(1,1) will be the sum ...

  A(1,1)B(1,1) +A(1,2)B(2,1) +A(1,3)B(3,1)

Calculators, apps, spreadsheets, and web sites are available that will perform this arithmetic for you. It can be a bit tedious to do by hand.

Here the product is ...

  [tex]A\cdot B=\left[\begin{array}{ccc}2&3&-1\\0&2&5\\2&4&0\end{array}\right] \cdot\left[\begin{array}{c}2&1&2\end{array}\right] =\left[\begin{array}{c}2(2)+3(1)+(-1)(2)&0(2)+2(1)+5(2)&2(2)+4(1)+0(2)\end{array}\right] \\\\=\left[\begin{array}{c}5&12&8\end{array}\right][/tex]

21. 13/4 x 42/9 =
O
A. 132/18
B. 64/9
O
C. 77/18
D. 41/6
Worth 2 points

Answers

Hi! 13/4 X 42/9 = 91/6 or 91/6 ls there any other options to choose from?

more math questions if you would

Answers

Answer:

A.

Step-by-step explanation:

So we are given the function:

[tex]f(x)=7x+8[/tex]

To find the inverse of the function, we simply need to flip f(x) and x and then solve for f(x). Thus:

[tex]x=7f^{-1}(x)+8\\x-8=7f^{-1}(x)\\f^{-1}(x)=\frac{x-8}{7}[/tex]

So the answer is A.

Answer:

[tex]\large \boxed{\mathrm{Option \ A}}[/tex]

Step-by-step explanation:

f(x) = 7x+8

Write f(x) as y.

y = 7x + 8

Switch variables.

x = 7y + 8

Solve for y to find the inverse.

x - 8 = 7y

[tex]\frac{x-8}{7}[/tex] = y

The points (-6,-4) and (3,5) are the endpoints of the diameter of a circle. Find the length of the radius of the circle.
The length of the radius is a
(Round to the nearest hundredth as needed.)

Answers

Answer:

40.5

Step-by-step explanation:

diameter^2 = (3 +6)^2 + (5+4)^2

or, d^2 = 9^2 + 9^2

or, d^2 = 81 +81

or,d^2 =162

or d=√ 162

• d= 81

then radius = d/2

r = 81/2

•r= 40.5 ans

BRAINLIEST ANSWER GIVEN! Find the equation of the line passing through the pair points (-8,6) (-9,-9). The equation of the line in the form is Ax+By=C.

Answers

Answer:

y=15x+126

Step-by-step explanation:

the slope is

15 because -8-(-9) is 1 and 6-(-9) is 15 and y is over x so slope 15

To find y intercept start from -8,6 and add 15 to the y value every time you add one to the x value

you will add 8 times and you get 126 as the intercept

In tests of a computer component, it is found that the mean time between failures is 520 hours. A modification is made which is supposed to increase the time between failures. Tests on a random sample of 10 modified components resulted in the following times (in hours) between failures. 518 548 561 523 536 499 538 557 528 563 At the 0.05 significance level, test the claim that for the modified components, the mean time between failures is greater than 520 hours. Use the P-value method of testing hypotheses.
H0:
H1:
Test Statistic:
Critical Value:
Do you reject H0?
Conclusion:
If you were told that the p-value for the test statistic for this hypothesis test is 0.014, would you reach the same decision that you made for the Rejection of H0 and the conclusion as above?

Answers

Answer:

As the calculated value of t is greater than critical value reject H0. The tests supports the claim at ∝= 0.05

If the p-value for the test statistic for this hypothesis test is 0.014, then the critical region is t ( with df=9) for a right tailed test is 2.821

then we would accept H0. The test would not support the claim at ∝= 0.01

Step-by-step explanation:

Mean x`= 518 +548 +561 +523 + 536 + 499+  538 + 557+ 528 +563 /10

x`= 537.1

The Variance is  = 20.70

H0 μ≤ 520

Ha μ > 520

Significance level is set at ∝= 0.05

The critical region is t ( with df=9) for a right tailed test is 1.8331

The test statistic under H0 is

t=x`- x/ s/ √n

Which has t distribution with n-1 degrees of freedom which is equal to 9

t=x`- x/ s/ √n

t = 537.1- 520 / 20.7 / √10

t= 17.1 / 20.7/ 3.16227

t= 17.1/ 6.5459

t= 2.6122

As the calculated value of t is greater than critical value reject H0. The tests supports the claim at ∝= 0.05

If the p-value for the test statistic for this hypothesis test is 0.014, then the critical region is t ( with df=9) for a right tailed test is 2.821

then we would accept H0. The test would not support the claim at ∝= 0.01

A plane took off at a point that is 42 meters from the control tower. The flight path takes the plane over the control tower that is 98 meters high. After traveling 83 meters, which statement is most accurate?

A. The plane needs to be about 15 meters higher to clear the tower.
B. The plane clears the tower with about 27 meters to spare.
C. The plane clears the tower with about 15 meters to spare.
D. The plane needs to be about 27 meters higher to clear the tower.

Answers

Answer:

D. The plane needs to be about 27 meters higher to clear the tower.

Step-by-step explanation:

In this scenario a triangle is being formed. The base the plane's takeoff point to the tower base which is 42 meters (x).

The hypothenus is the distance travelled by the plane which is 83 meters (h)

The height of the tower is 98 Meters

We want to calculate the height of our triangle (y) so we can guage if the plane scaled the tower.

According to Pythagorean theorem

(x^2) + (y^2) = h^2

y = √ (h^2) - (x^2)

y = √ (83^2) - (42^2)

y= √(6889 - 1764)

y= 71.59 Meters

The height from the plane's position to the top of the tower will be

Height difference = 98 - 71.59 = 26.41 Meters

So the plane should go about 27 Meters higher to clear the tower

A train is running at the speed of 90 mph. The length of the train is 300 ft. How long would it take to cross a railway platform 492 ft long?

Answers

Answer:

Time = 1.45152 seconds

Step-by-step explanation:

1 foot = 0.000189 mile

300 ft = 300*0.000189

300 ft = 0.0567 miles

492 ft = 492*0.000189

492 ft = 0.092988 miles

Distance left to be covered by the train

= 0.092988-0.0567

= 0.036288 miles

Speed= 90mph

Time taken = distance/speed

Time taken= 0.036288/90

Time = 4.032*10^-4 hour

Time = 4.032*10^-4*60*60

Time = 1.45152 seconds

Identify the decimals labeled with the letters A, B, and C on the scale below. Letter A represents the decimal Letter B represents the decimal Letter C represents the decimal

Answers

[tex]10[/tex] divisions between $389$ and $390$ so each division is $\frac{390-389}{10}=0.1$

A is 8 division from $389$, so, A is $389+8\times 0.1=389.8$

similarly, C is one division behind $389$ so it is $389-1\times 0.1=388.9$

and B is $390.3$

Solve the following equations
x-1=6/x​

Answers

[tex]x-1=\dfrac{6}{x}\qquad(x\not=0)\\\\x^2-x=6\\x^2-x-6=0\\x^2+2x-3x-6=0\\x(x+2)-3(x+2)=0\\(x-3)(x+2)=0\\x=3 \vee x=-2[/tex]

Points A( − 1, 7), B(2, 19), and C(3, y) are on the same line. Find y.

Answers

Answer: y=23

Step-by-step explanation:

If Points and A,B and C lines on the same line then they will have the same slopes so since we have the coordinates of  A and B we will use the to write an equation in slope intercept form.

To write it in slope intercept form we will need to find the slope and the y intercept.

To find the slope you will find the change in the y coordinates and divide it by the change in the x coordinates.

Using the coordinates (-1,7) and (2,19)  the y coordinates are 7 and 19 and the x coordinates are -1 and 2.

Slope :  [tex]\frac{7-19}{-1-2} \frac{-12}{-3} = 4[/tex]   In this case the slope is 4 so we will use that to find the y intercept by using point A coordinate.

  The slope intercept formula says that y=mx +b  where me is the slope and b is the y intercept.

 7=4(-1) + b  

7 = -4 + b

+4   +4

 b= 11       The y intercept is 11.

Now we can write the whole equation as y=4x + 11   .

To answer the question now, where we need to find y , we will plot the x coordinate which is 3 into the equation and solve for y.

 y = 4(3) + 11  

y = 12 + 11

 y = 23

What is the diameter of the base of the cone below, to the nearest foot, if the volume is 314 cubic feet? Use π = 3.14.

Answers

This question is incomplete because it lacks the required diagram. Please find attached the diagram required to answer the question.

Answer:

14 feet

Step-by-step explanation:

The volume of a cone = 1/3 πr²h

In the above question, we are given the volume = 314 cubic feet

the height is given in the attached diagram = 6ft

Step 1

We find the radius.

From the formula for the volume of a cone, we can derive the formula for radius of the cone.

Radius of the cone = √(3 × V/π × h

π = 3.14

Radius of the cone = √( 3 × 314 /3.14 × 6

Radius of the cone = √942/3.14× 6

Radius = √50

= 7.0710678119feet

Step 2

Diameter of the cone = Radius of the cone × 2

= 7.0710678119 × 2

= 14.142135624 feet

Approximately to the nearest foot = 14 feet

Therefore, the diameter of the cone to the nearest foot = 14 feet.

Write each percent as a fraction in simple form 15 3/5%

Answers

Answer:

39/250

Step-by-step explanation:

15 3/5 = 78/5 and percent means per 100, so

(78/5)/100 = (78/5)(1/100)

=78/500 = 39/250

The statistics (U or U') used in the Mann-Whitney U test, measure _________. Group of answer choices the separation between the two groups the direction of the differences between pairs of scores the power of the experiment the differences between the means of the two groups

Answers

Statistics U or U' in the Mann-Whitney U test, measure the differences between the means of the two groups

In a test with two groups, the smaller value between the statistics U and U' points to the research hypothesis, while the larger value points to the alternative hypothesis.

The formula to calculate U and U' is:

[tex]U = n_1 \times n_2 + \frac{n_1(n_1+1)}{2} - R_1[/tex]

[tex]U' = n_1 \times n_2 + \frac{n_2(n_2+1)}{2} - R_2[/tex]

Take, for instance;

The values of U and U' in a test where the research hypothesis of two populations are not equal are:

[tex]U = 0[/tex]

[tex]U' = 22[/tex]

Recall that, the smallest of the 2 value supports the research hypothesis.

This means that [tex]U = 0[/tex] shows that the difference in the population is 0.

Read more at:

https://brainly.com/question/17905876

will rate7 you brainliest

Answers

Answer:

[tex]\Large \boxed{\sf \bf \ \ \dfrac{x^2-x-6}{x^2-3x+2} \ \ }[/tex]

Step-by-step explanation:

Hello, first of all, we will check if we can factorise the polynomials.

[tex]\boxed{x^2+6x+8}\\\\\text{The sum of the zeroes is -6=(-4)+(-2) and the product 8=(-4)*(-2), so}\\\\x^2+6x+8=x^2+2x+4x+8=x(x+2)+4(x+2)=(x+2)(x+4)[/tex]

[tex]\boxed{x^2+3x-10}\\\\\text{The sum of the zeroes is -3=(-5)+(+2) and the product -10=(-5)*(+2), so}\\\\x^2+3x-10=x^2+5x-2x-10=x(x+5)-2(x+5)=(x+5)(x-2)[/tex]

[tex]\boxed{x^2+2x-15}\\\\\text{The sum of the zeroes is -2=(-5)+(+3) and the product -15=(-5)*(+3), so}\\\\x^2+2x-15=x^2-3x+5x-15=x(x-3)+5(x-3)=(x+5)(x-3)[/tex]

[tex]\boxed{x^2+3x-4}\\\\\text{The sum of the zeroes is -3=(-4)+(+1) and the product -4=(-4)*(+1), so}\\\\x^2+3x-4=x^2-x+4x-4=x(x-1)+4(x-1)=(x+4)(x-1)[/tex]

Now, let's compute the product.

[tex]\dfrac{x^2+6x+8}{x^2+3x-10}\cdot \dfrac{x^2+2x-15}{x^2+3x-4}\\\\\\=\dfrac{(x+2)(x+4)}{(x+5)(x-2)}\cdot \dfrac{(x+5)(x-3)}{(x+4)(x-1)}\\\\\\\text{We can simplify}\\\\=\dfrac{(x+2)}{(x-2)}\cdot \dfrac{(x-3)}{(x-1)}\\\\\\=\large \boxed{\dfrac{x^2-x-6}{x^2-3x+2}}[/tex]

So the correct answer is the first one.

Thank you.

Other Questions
11z/(z+3) = 13 - i , zC please solve it The shape of American government is based mostly on which of the following English political ideas:Atotalitarian governmentB.limited governmentparliamentarian governmentOD.Constitutional Monarchy government A ball is ejected to the right with an unknown horizontal velocity from the top of a pillar that is 50 meters in height. At the exact instant, a carriage moving on rails is also released to the right from the bottom of the pillar. Calculate the velocity with which the carriage should be released so that the ball falls in the carriage after the carriage has traveled a distance of 50 meters on the ground.A. 12.20 meters/ seconds B. 13.23 meters/ seconds C. 14.30 meters/ seconds D. 15.65 meters/ seconds** E. 16.00 meters/ seconds I believe D is right = 15.65m/s Also ** on Plato You have a Nano Server named Nano1. Which cmdlet should you use to identify whether the DNS Server role is installed on Nano1 Find the length of MK A. 7 B. 47 C. 19 D. 34 Amir throws a stone off of a bridge into a river. The stone's height (in meters above the water) ttt seconds after Amir throws it is modeled by h(t)=-5t^2+20t+160h(t)=5t 2 +20t+160h, left parenthesis, t, right parenthesis, equals, minus, 5, t, squared, plus, 20, t, plus, 160 Amir wants to know when the stone will reach its highest point. 1) Rewrite the function in a different form (factored or vertex) where the answer appears as a number in the equation. h(t)=h(t)=h, left parenthesis, t, right parenthesis, equals 2) How many seconds after being thrown did the stone reach its highest point? what type of molecule do plant cells use for long term energy storage Please answer this question now A rectangle's length and width are in a ratio of 3:1. The perimeter is 72 inches. What are the length and width? Which of the following is found in the 'rungs' of a DNA strand?A. UracilB. ThymineC. Deoxy RiboseD. Phosphate Youre given two side lengths of 10 centimeters and 8 centimeters. The angle between the sides measures 40. How many triangles can you construct using these measurements? How do we derive the sum rule in differentiation? (ie. (u+v)' = u' + v') Secure email technique that provides confidentiality, along with authentication, integrity, and non-repudiation. It uses a centralized trust model where users get a certificate from a trusted CA.a. Trueb. False what mass of aluminum nitrate do you need to prepare 3.58L of a 1.77M Solution? Chester's balance sheet has $105,038,000 in equity. Further, the company is expecting net income of 3,000,000 next year, and also expecting to issue $4,000,000 in new stock. If there are no dividends paid what will beChester's book value? 3. If the coordinates of the two points 1 pointare P(-7,5) and Q (-6, 9), then(abscissa of P) - (abscissa of Q) is.... *a) 3O b) 1c) -2O d) -1Other: P-weight blocks D and E are connected by the rope which passes through pulley B and are supported by the isorectangular prism articulated to the ground at its vertex A, while vertex C is attached to the vertical cord fixed to the ground. If the coefficient of friction between the prism and the blocks is 0.4; determine the maximum angle that measures the inclination of the AC face with respect to the horizontal so that the system remains in equilibrium. find the surface area of the composite figure. answer asap please :) Your customer, age 60, is retired and living at home with a fully paid-off mortgage. Her portfolio contains growth stocks and high-quality bonds, and she is a long-time investor and comfortable with moderate risk. Her objective is a moderate level of current income to supplement her corporate pension plan distributions and the earnings from her traditional IRA. How are the distributions taxed from her IRA HELLLLLLPPOP HELPPPP